Renaud11232.github.io

View on GitHub

Access Denied

~$ cd ..

Author: BorelEnzo

We were given the following bash script:

#!/bin/bash
TOTAL=$(cat $0 | wc -l)
TAIL=7
HEAD=$(($TOTAL-$TAIL-1))
CHECKSUM=$({ head $0 -n $HEAD && tail $0 -n $TAIL; } | md5sum | cut -d ' ' -f 1)

if [ "$CHECKSUM" != "67264de22f36c95c203074971ad1da69" ]; then exit; fi
ENCRYPTED="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"

DECRYPTED=$(echo $ENCRYPTED | openssl enc -aes-128-cbc -a -d -salt -pass pass:$CHECKSUM -A)
if [ "$USER" != "deploymentPC1" ]; then exit; fi
echo $DECRYPTED | base64 -d > input.elf
objcopy -O ihex input.elf output.hex
avrdude -p atmega328p -b 115200 -P /dev/ttyS10 -U output.hex -c arduino
rm input.elf

We didn’t have any Arduino at hand, but it was not necessary.

Decrypt the binary

First task was then to decrypt the binary using the given password. We used Python to un-base64 the encrypted file:

import base64
f = open("cipher.txt", "wb")
f.write(base64.b64decode("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"))
f.close()

It’s indeed recognized as an openssl encrypted file:

file cipher.txt
	cipher.txt: openssl enc'd data with salted password

We then tried to decrypt with the following command:

$ openssl enc -aes-128-cbc -d -in cipher.txt -k 67264de22f36c95c203074971ad1da69

but received the following error:

bad decrypt
139766350381120:error:06065064:digital envelope routines:EVP_DecryptFinal_ex:bad decrypt:../crypto/evp/evp_enc.c:541:

By googling, we found that it was a known bug, and that a workaround exists:

bug

We tried again with the following command:

$ openssl enc -aes-128-cbc -d -in cipher.txt -k 67264de22f36c95c203074971ad1da69 -md md5 | base64 -d > update.elf
$ file update.elf
	update.elf: ELF 32-bit LSB executable, Atmel AVR 8-bit, version 1 (SYSV), statically linked, not stripped

It was then time to reverse it!

Reverse the binary

The second part was actually easier than what we supposed at the beginning, because we didn’t have to execute the program. A static analysis was sufficient to get the flag. By loading the binary in radare2, we found the following main routine:

[0x00000000]> sf sym.main
[0x00001b14]> pdf
/ (fcn) sym.main 30
|   sym.main ();
|           ; CALL XREF from 0x000000b2 (entry0)
|           0x00001b14      cf93           push r28
|           0x00001b16      df93           push r29
|           0x00001b18      cdb7           in r28, 0x3d                ; IO SPL: Stack lower bits SP0-SP7 ; '='
|           0x00001b1a      deb7           in r29, 0x3e                ; IO SPH: Stack higher bits SP8-SP10 ; '>'
|           0x00001b1c      0e94200f       call sym.init
|           0x00001b20      0e947c0d       call sym.initVariant
\           0x00001b24      0e948100       call sym.setup
|           ; JMP XREF from 0x00001b30 (sym.main)
|       .-> 0x00001b28      0e949000       call sym.loop
|       |   0x00001b2c      0e946901       call sym.serialEventRun
|       `=< 0x00001b30      fbcf           rjmp 0x1b28

We tried to disassemble several functions, and in sym.loop, we found some interesting instructions:

[0x00000102]> sf sym.loop
[0x00000120]> pdf
/ (fcn) sym.loop 402
|   sym.loop ();
|           ; CALL XREF from 0x00001b28 (sym.main)
|           0x00000120      0f93           push r16
|           0x00000122      1f93           push r17
|           0x00000124      cf93           push r28
|           0x00000126      df93           push r29
|           0x00000128      cdb7           in r28, 0x3d                ; IO SPL: Stack lower bits SP0-SP7 ; '='
|           0x0000012a      deb7           in r29, 0x3e                ; IO SPH: Stack higher bits SP8-SP10 ; '>'
|           0x0000012c      60e2           ldi r22, 0x20
|           0x0000012e      71e0           ldi r23, 0x01
|           0x00000130      81e4           ldi r24, 0x41
|           0x00000132      92e0           ldi r25, 0x02
|           0x00000134      0e94d109       call fcn.000013a2
|           0x00000138      6ee3           ldi r22, 0x3e
|           0x0000013a      71e0           ldi r23, 0x01
|           0x0000013c      81e4           ldi r24, 0x41
|           0x0000013e      92e0           ldi r25, 0x02
|           0x00000140      0e94d109       call fcn.000013a2
|           0x00000144      10923f02       sts 0x23f, r1
|           0x00000148      81e4           ldi r24, 0x41
|           0x0000014a      92e0           ldi r25, 0x02
|           0x0000014c      0e94be03       call sym.HardwareSerial::flush
|           0x00000150      68ec           ldi r22, 0xc8
|           0x00000152      70e0           ldi r23, 0x00
|           0x00000154      80e0           ldi r24, 0x00
|           0x00000156      90e0           ldi r25, 0x00
|           0x00000158      0e94960e       call sym.delay
|       ,=< 0x0000015c      21c0           rjmp 0x1a0
|      .--> 0x0000015e      81e4           ldi r24, 0x41
|      ||   0x00000160      92e0           ldi r25, 0x02
|      ||   0x00000162      0e940103       call sym.HardwareSerial::available
|      ||   0x00000166      21e0           ldi r18, 0x01
|      ||   0x00000168      1816           cp r1, r24
|      ||   0x0000016a      1906           cpc r1, r25
|     ,===< 0x0000016c      0cf0           brlt 0x170
|     |||   0x0000016e      20e0           ldi r18, 0x00
|     `---> 0x00000170      2223           tst r18
|     ,===< 0x00000172      b1f0           breq 0x1a0
|     |||   0x00000174      80913f02       lds r24, 0x23f
|     |||   0x00000178      882f           mov r24, r24
|     |||   0x0000017a      90e0           ldi r25, 0x00
|     |||   0x0000017c      880f           lsl r24
|     |||   ; JMP XREF from 0x00002154 (sym.__fixunssfsi)
|     |||   0x0000017e      991f           rol r25
|     |||   0x00000180      8e5f           subi r24, 0xfe
|     |||   0x00000182      9e4f           sbci r25, 0xfe
|     |||   0x00000184      fc01           movw r30, r24
|     |||   0x00000186      0081           ld r16, z
|     |||   0x00000188      1181           ldd r17, z+1
|     |||   0x0000018a      81e4           ldi r24, 0x41
|     |||   0x0000018c      92e0           ldi r25, 0x02
|     |||   0x0000018e      0e944e03       call sym.HardwareSerial::read; ssize_t read(int fildes, void *buf, size_t nbyte);
|     |||   0x00000192      f801           movw r30, r16
|     |||   0x00000194      8083           std z+0, r24
|     |||   0x00000196      80913f02       lds r24, 0x23f
|     |||   0x0000019a      8f5f           subi r24, 0xff
|     |||   0x0000019c      80933f02       sts 0x23f, r24
|     |||   ; JMP XREF from 0x0000015c (sym.loop)
|     `-`-> 0x000001a0      80913f02       lds r24, 0x23f
|      |    0x000001a4      282f           mov r18, r24
|      |    ; JMP XREF from 0x00002192 (sym.__floatunsisf)
|      |    0x000001a6      30e0           ldi r19, 0x00
|      |    0x000001a8      80910001       lds r24, 0x100
|      |    0x000001ac      90910101       lds r25, 0x101
|      |    0x000001b0      2817           cp r18, r24
|      |    0x000001b2      3907           cpc r19, r25
|      `==< 0x000001b4      a4f2           brlt 0x15e
|           0x000001b6      10924002       sts 0x240, r1
|           0x000001ba      0027           clr r16
|           ;-- checkByte0:
|           0x000001bc      1fe4           ldi r17, 0x4f
|           0x000001be      20913002       lds r18, 0x230
|           0x000001c2      1227           eor r17, r18
|           0x000001c4      1c30           cpi r17, 0x0c
|       ,=< 0x000001c6      09f4           brne loc.checkByte1
|       |   0x000001c8      0395           inc r16
|       `-> ;-- checkByte1:
|       `-> 0x000001ca      11e5           ldi r17, 0x51
|           0x000001cc      20913102       lds r18, 0x231
|           0x000001d0      1227           eor r17, r18
|           0x000001d2      1230           cpi r17, 0x02
|       ,=< 0x000001d4      09f4           brne loc.checkByte2
|       |   0x000001d6      0395           inc r16
|       `-> ;-- checkByte2:
|       `-> 0x000001d8      17e4           ldi r17, 0x47
|           0x000001da      20913202       lds r18, 0x232
|           0x000001de      1227           eor r17, r18
|           0x000001e0      1430           cpi r17, 0x04
|       ,=< 0x000001e2      09f4           brne loc.checkByte3
|       |   0x000001e4      0395           inc r16
|       `-> ;-- checkByte3:
|       `-> 0x000001e6      19e3           ldi r17, 0x39
|           0x000001e8      20913302       lds r18, 0x233
|           0x000001ec      1227           eor r17, r18
|           0x000001ee      1234           cpi r17, 0x42
|       ,=< 0x000001f0      09f4           brne loc.checkByte4
|       |   0x000001f2      0395           inc r16
|       `-> ;-- checkByte4:
|       `-> 0x000001f4      13e6           ldi r17, 0x63
|           0x000001f6      20913402       lds r18, 0x234
|           0x000001fa      1227           eor r17, r18
|           0x000001fc      1230           cpi r17, 0x02
|       ,=< 0x000001fe      09f4           brne loc.checkByte5
|       |   0x00000200      0395           inc r16
|       `-> ;-- checkByte5:
|       `-> 0x00000202      1fe6           ldi r17, 0x6f
|           0x00000204      20913502       lds r18, 0x235
|           0x00000208      1227           eor r17, r18
|           0x0000020a      1933           cpi r17, 0x39
|       ,=< 0x0000020c      09f4           brne loc.checkByte6
|       |   0x0000020e      0395           inc r16
|       `-> ;-- checkByte6:
|       `-> 0x00000210      18e6           ldi r17, 0x68
|           0x00000212      20913602       lds r18, 0x236
|           0x00000216      1227           eor r17, r18
|           0x00000218      1a31           cpi r17, 0x1a
|       ,=< 0x0000021a      09f4           brne loc.checkByte7
|       |   0x0000021c      0395           inc r16
|       `-> ;-- checkByte7:
|       `-> 0x0000021e      1fe7           ldi r17, 0x7f
|           0x00000220      20913702       lds r18, 0x237
|           0x00000224      1227           eor r17, r18
|           0x00000226      1032           cpi r17, 0x20
|       ,=< 0x00000228      09f4           brne loc.checkByte8
|       |   0x0000022a      0395           inc r16
|       `-> ;-- checkByte8:
|       `-> 0x0000022c      15e6           ldi r17, 0x65
|           0x0000022e      20913802       lds r18, 0x238
|           0x00000232      1227           eor r17, r18
|           0x00000234      1935           cpi r17, 0x59
|       ,=< 0x00000236      09f4           brne loc.checkByte9
|       |   0x00000238      0395           inc r16
|       `-> ;-- checkByte9:
|       `-> 0x0000023a      16e6           ldi r17, 0x66
|           0x0000023c      20913902       lds r18, 0x239
|           0x00000240      1227           eor r17, r18
|           0x00000242      1535           cpi r17, 0x55
|       ,=< 0x00000244      09f4           brne loc.checkByte10
|       |   0x00000246      0395           inc r16
|       `-> ;-- checkByte10:
|       `-> 0x00000248      1ae6           ldi r17, 0x6a
|           0x0000024a      20913a02       lds r18, loc.checkByte9
|           0x0000024e      1227           eor r17, r18
|           0x00000250      1533           cpi r17, 0x35
|       ,=< 0x00000252      09f4           brne loc.checkByte11
|       |   0x00000254      0395           inc r16
|       `-> ;-- checkByte11:
|       `-> 0x00000256      18e7           ldi r17, 0x78
|           0x00000258      20913b02       lds r18, 0x23b
|           0x0000025c      1227           eor r17, r18
|           0x0000025e      1933           cpi r17, 0x39
|       ,=< 0x00000260      09f4           brne loc.checkByte12
|       |   0x00000262      0395           inc r16
|       `-> ;-- checkByte12:
|       `-> 0x00000264      1fe3           ldi r17, 0x3f
|           0x00000266      20913c02       lds r18, 0x23c
|           0x0000026a      1227           eor r17, r18
|           0x0000026c      1c34           cpi r17, 0x4c
|       ,=< 0x0000026e      09f4           brne loc.checkByte13
|       |   0x00000270      0395           inc r16
|       `-> ;-- checkByte13:
|       `-> 0x00000272      16e4           ldi r17, 0x46
|           0x00000274      20913d02       lds r18, 0x23d
|           0x00000278      1227           eor r17, r18
|           0x0000027a      1b30           cpi r17, 0x0b
|       ,=< 0x0000027c      09f4           brne loc.checkByte14
|       |   0x0000027e      0395           inc r16
|       `-> ;-- checkByte14:
|       `-> 0x00000280      14e5           ldi r17, 0x54
|           0x00000282      20913e02       lds r18, 0x23e
|           0x00000286      1227           eor r17, r18
|           0x00000288      1932           cpi r17, 0x29
|       ,=< 0x0000028a      09f4           brne loc.noMore
|       |   0x0000028c      0395           inc r16
|       `-> ;-- noMore:
|       `-> 0x0000028e      0f30           cpi r16, 0x0f
|       ,=< 0x00000290      19f0           breq loc.passOK
|       |   0x00000292      0027           clr r16
|      ,==< 0x00000294      0c944d01       jmp loc.done
|      |`-> ;-- passOK:
|      |`-> 0x00000298      01e0           ldi r16, 0x01
|      `--> ;-- done:
|      |    ; JMP XREF from 0x00000294 (sym.loop)
|      `--> 0x0000029a      00934002       sts 0x240, r16
|           0x0000029e      90914002       lds r25, 0x240
|           0x000002a2      81e0           ldi r24, 0x01
|           0x000002a4      9130           cpi r25, 0x01
|       ,=< 0x000002a6      09f0           breq 0x2aa
|       |   0x000002a8      80e0           ldi r24, 0x00
|       `-> 0x000002aa      8823           tst r24
|       ,=< 0x000002ac      39f0           breq 0x2bc                  ; sym.__subsf3-0x1cae
|       |   0x000002ae      6fe4           ldi r22, 0x4f
|       |   ; JMP XREF from 0x000022ca (unk)
\       |   0x000002b0      71e0           ldi r23, 0x01

Each character is XORed with a constant and compared against another constant, in each checkByteX and if all tests pass, a jump to passOK is done.

Each constrain is built as follows:

The constraints are as follows:

and finally we got CSC{aVr_<3_AsM}